LSAT and Law School Admissions Forum

Get expert LSAT preparation and law school admissions advice from PowerScore Test Preparation.

 Administrator
PowerScore Staff
  • PowerScore Staff
  • Posts: 8917
  • Joined: Feb 02, 2011
|
#35322
Complete Question Explanation

Justify the Conclusion. The correct answer choice (E)

In this stimulus the author presents the following basic argument:
  • Premise: Most Vista Arms upper floor apartments have scenic views.

    Conclusion: Therefore the apartment building has at least one studio apartment with scenic
    views.
The author appears to have jumped to this conclusion based on insufficient information; all that we
know is that the majority of the apartments on the upper floors have nice views, yet the author’s
conclusion deals with studio apartments. The stimulus is followed by a Justify question, and the
answer choice that justifies the author’s conclusion must somehow deal with this new, rogue element.

Answer choice (A): This choice provides that all lower floor apartments have scenic views, but there
is no way to know whether there are any studio apartments on the lower floors. This choice says
nothing about studio apartments, so it cannot justify the author’s conclusion.

Answer choice (B): This choice might look enticing at first, because it deals with all of the
apartments in the building, but there is no way to know whether The Vista Arms apartments have
any studio apartments at all. Since this choice does not deal with the conclusion’s rogue element, it
cannot be the correct answer choice to this Justify question.

Answer choice (C): If you found this choice appealing, it might be because this answer does deal
with studio apartments, the rogue element from the author’s conclusion. This choice provides that
most of the building’s apartments are studios, but with the limited information from this stimulus
(that the majority of apartments on the upper floors have scenic views), this answer is not quite
sufficient to justify the author’s conclusion.

If, for example 70 percent of the apartments are studio apartments, the remaining 30 percent (let’s
call those the top three floors of a ten story building) might be mostly scenic, while the studio
apartments on the bottom seven floors might all be windowless. Since this choice would not justify
the conclusion that at least one studio has a nice view, this cannot be the right answer choice to this
Justify question.

Answer choice (D): If there were only one studio apartment, located in the basement, then this
choice would not justify the conclusion that at least one of the apartments is a studio with a view.

Answer choice (E): This is the correct answer choice. The term “most” means more than half. If
more than half of the upper floor apartments have scenic views, and, as this choice provides, more
than half of upper floor apartments are studios, this guarantees at least some overlap between the two
attributes. In other words, if 51 out of 100 upper floor apartments have scenic views, and 51 out of
those 100 upper floor apartments are studios, there must be some studios with scenic views:
  • Premise: Most Vista Arms upper floor apartments have scenic views.
    + Answer Choice (E): Most upper floor apartments are studios.
    Justified conclusion: There is at least one studio on an upper floor with a view.
 lsatstudier
  • Posts: 49
  • Joined: Oct 24, 2016
|
#30631
Hi,

Could someone explain this one as well? I think I understand why the correct answer is E, but I am not confident on my ability to find the correct answer in the short amount of time I have.

Thank you!
User avatar
 Jonathan Evans
PowerScore Staff
  • PowerScore Staff
  • Posts: 726
  • Joined: Jun 09, 2016
|
#30778
LSATStudier,

This is a Justify the Conclusion problem. You must provide information sufficient (when added to the premises) to guarantee the validity of the conclusion. So far in this problem we know:

P1: apartments on upper floors of building :most: scenic views

C: studio apartment in building :some: scenic views

We need to find something to combine with P1 to guarantee the validity of the conclusion C. You need something that tells you that:

apartments on upper floors of building :most: studio apartment

You have to make sure that there is an intersection between the "apartments on upper floors of building with scenic views" and the "studio apartments in building," that there is at least one example of an "apartment on upper floors of building with a scenic view" that is also a "studio apartment." If you ensure that most of the "apartments on upper floors of building" are "studio apartments," then there must exist at least one "studio apartment" with a "scenic view."
 rahimlsat
  • Posts: 8
  • Joined: Oct 24, 2018
|
#60295
Isn't this formal logic?

I had:

Premise: Auf :most: scenic view

Conclusion: studio apts :some: scenic view

E says:

Auf :most: studio apts

Combine the premise and E makes Auf in the middle with two :most: going away from it. One for studio apts and one for scenic view. The two most arrows moving away from the same thing means that those two things on the ends can be linked with a :some:
User avatar
 Dave Killoran
PowerScore Staff
  • PowerScore Staff
  • Posts: 5852
  • Joined: Mar 25, 2011
|
#60344
rahimlsat wrote:Isn't this formal logic?

I had:

Premise: Auf :most: scenic view

Conclusion: studio apts :some: scenic view

E says:

Auf :most: studio apts

Combine the premise and E makes Auf in the middle with two :most: going away from it. One for studio apts and one for scenic view. The two most arrows moving away from the same thing means that those two things on the ends can be linked with a :some:
Yes it is, and we have it classified that way in our test analysis program. Once it's all pieced together, it's the classic "two "Mosts" leading away from a common source lead to a "some" conclusion" setup as you correctly outlined :-D Good job!

Thanks!
 akshanye
  • Posts: 3
  • Joined: Aug 03, 2019
|
#71650
Okay so In the book (2018 publication) it says two most leading away does not produce any inference (Pg. 453). Is the book wrong?
 Jeremy Press
PowerScore Staff
  • PowerScore Staff
  • Posts: 1000
  • Joined: Jun 12, 2017
|
#71692
Hi akshanye,

The book has got it right on this issue, and you've misread what is there! There is one circumstance where two "most" statements can produce an additive "some" inference, and it's the one that page 453 of the 2018 LRB refers to when it states, "This inference can only occur when the two mosts each "lead away" from the middle variable."

The example given in the LRB is "Most B's are A's," in combination with the statement "Most B's are C's." The middle variable here would be B, and in our diagramming system, an arrow would "lead away" from the B on one side to A, and "lead away" from the B on the other side to C, like so:
Screen Shot 2019-11-04 at 12.29.17 PM.png
In this question, the two most statements are the premise in the stimulus, "Most Vista Arms upper floor apartments have scenic views," and answer choice E, "Most Vista Arms upper floor apartments are studios." Those statements would diagram as such:
Screen Shot 2019-11-04 at 12.29.24 PM.png
In each case, the "outer links" of the chains can be joined by a "some," i.e. "Some A's are C's," and "Some apartments with scenic views are studios."

I hope this helps!

Jeremy
You do not have the required permissions to view the files attached to this post.
 lsatstudying11
  • Posts: 54
  • Joined: Jul 30, 2020
|
#77606
Hi!

Would it also have been possible for the correct answer to be super strong and be an all statement rather than a most? I was thinking that the right answer would either be 'most upper floor apartments are studios' or 'all upper floor apartments are studios.'

Thank you in advance for your help! 8-)
 Jeremy Press
PowerScore Staff
  • PowerScore Staff
  • Posts: 1000
  • Joined: Jun 12, 2017
|
#77685
Hi lsatstudying,

Yes, that would have been fine as well! A Justify question answer can be stronger than necessary to justify the conclusion--it's still correct, so long as it is strong enough to render the conclusion valid. Your proposed answer (All upper floor apartments are studios) would work, because it renders the conclusion valid.

Keep up the great work!

Jeremy

Get the most out of your LSAT Prep Plus subscription.

Analyze and track your performance with our Testing and Analytics Package.